2013 AIME I Problems/Problem 15

Revision as of 16:32, 12 February 2016 by FractalMathHistory (talk | contribs) (Solution)

Problem 15

Let $N$ be the number of ordered triples $(A,B,C)$ of integers satisfying the conditions (a) $0\le A<B<C\le99$, (b) there exist integers $a$, $b$, and $c$, and prime $p$ where $0\le b<a<c<p$, (c) $p$ divides $A-a$, $B-b$, and $C-c$, and (d) each ordered triple $(A,B,C)$ and each ordered triple $(b,a,c)$ form arithmetic sequences. Find $N$.

Solution

From condition (d), we have $(A,B,C)=(B-D,B,B+D)$ and $(b,a,c)=(a-d,a,a+d)$. Condition $\text{(c)}$ states that $p\mid B-D-a$, $p|B-a+d$, and $p\mid B+D-a-d$. We subtract the first two to get $p\mid-d-D$, and we do the same for the last two to get $p\mid 2d-D$. We subtract these two to get $p\mid 3d$. So $p\mid 3$ or $p\mid d$. The second case is clearly impossible, because that would make $c=a+d>p$, violating condition $\text{(b)}$. So we have $p\mid 3$, meaning $p=3$. Condition $\text{(b)}$ implies that $(b,a,c)=(0,1,2)$ or $(a,b,c)\in (1,0,2)\rightarrow (-2,0,2)\text{ }(D\equiv 2\text{ mod 3}). Now we return to condition$\text{(c)}$, which now implies that$(A,B,C)\equiv(-2,0,2)\pmod{3}$. Now, we set$B=3k$for increasing positive integer values of$k$.$B=0$yields no solutions.$B=3$gives$(A,B,C)=(1,3,5)$, giving us$1$solution. If$B=6$, we get$2$solutions,$(4,6,8)$and$(1,6,11)$. Proceeding in the manner, we see that if$B=48$, we get 16 solutions. However,$B=51$still gives$16$solutions because$C_\text{max}=2B-1=101>100$. Likewise,$B=54$gives$15$solutions. This continues until$B=96$gives one solution.$B=99$gives no solution. Thus,$N=1+2+\cdots+16+16+15+\cdots+1=2\cdot\frac{16(17)}{2}=16\cdot 17=\boxed{272}$.

See also

2013 AIME I (ProblemsAnswer KeyResources)
Preceded by
Problem 14
Followed by
Last Problem
1 2 3 4 5 6 7 8 9 10 11 12 13 14 15
All AIME Problems and Solutions

The problems on this page are copyrighted by the Mathematical Association of America's American Mathematics Competitions. AMC logo.png